Are you seeking one-on-one college counseling and/or essay support? Limited spots are now available. Click here to learn more.

How to Write the AP Lang Rhetorical Analysis Essay (With Example)

November 27, 2023

Feeling intimidated by the AP Lang Rhetorical Analysis Essay? We’re here to help demystify. Whether you’re cramming for the AP Lang exam right now or planning to take the test down the road, we’ve got crucial rubric information, helpful tips, and an essay example to prepare you for the big day. This post will cover 1) What is the AP Lang Rhetorical Analysis Essay? 2) AP Lang Rhetorical Analysis Rubric 3) AP Lang Rhetorical Analysis: Sample Prompt 4) AP Lang Rhetorical Analysis Essay Example 5)AP Lang Rhetorical Analysis Essay Example: Why It Works

What is the AP Lang Rhetorical Analysis Essay?

The AP Lang Rhetorical Analysis Essay is one of three essays included in the written portion of the AP English Exam. The full AP English Exam is 3 hours and 15 minutes long, with the first 60 minutes dedicated to multiple-choice questions. Once you complete the multiple-choice section, you move on to three equally weighted essays that ask you to synthesize, analyze, and interpret texts and develop well-reasoned arguments. The three essays include:

Synthesis essay: You’ll review various pieces of evidence and then write an essay that synthesizes (aka combines and interprets) the evidence and presents a clear argument. Read our write up on How to Write the AP Lang Synthesis Essay here.

Argumentative essay: You’ll take a stance on a specific topic and argue your case.

Rhetorical essay: You’ll read a provided passage, then analyze the author’s rhetorical choices and develop an argument that explains why the author made those rhetorical choices.

AP Lang Rhetorical Analysis Rubric

The AP Lang Rhetorical Analysis Essay is graded on just 3 rubric categories: Thesis, Evidence and Commentary, and Sophistication . At a glance, the rubric categories may seem vague, but AP exam graders are actually looking for very particular things in each category. We’ll break it down with dos and don’ts for each rubric category:

Thesis (0-1 point)

There’s nothing nebulous when it comes to grading AP Lang Rhetorical Analysis Essay thesis. You either have one or you don’t. Including a thesis gets you one point closer to a high score and leaving it out means you miss out on one crucial point. So, what makes a thesis that counts?

  • Make sure your thesis argues something about the author’s rhetorical choices. Making an argument means taking a risk and offering your own interpretation of the provided text. This is an argument that someone else might disagree with.
  • A good test to see if you have a thesis that makes an argument. In your head, add the phrase “I think that…” to the beginning of your thesis. If what follows doesn’t logically flow after that phrase (aka if what follows isn’t something you and only you think), it’s likely you’re not making an argument.
  • Avoid a thesis that merely restates the prompt.
  • Avoid a thesis that summarizes the text but does not make an argument.

Evidence and Commentary (0-4 points)

This rubric category is graded on a scale of 0-4 where 4 is the highest grade. Per the AP Lang Rhetorical Analysis rubric, to get a 4, you’ll want to:

  • Include lots of specific evidence from the text. There is no set golden number of quotes to include, but you’ll want to make sure you’re incorporating more than a couple pieces of evidence that support your argument about the author’s rhetorical choices.
  • Make sure you include more than one type of evidence, too. Let’s say you’re working on your essay and have gathered examples of alliteration to include as supporting evidence. That’s just one type of rhetorical choice, and it’s hard to make a credible argument if you’re only looking at one type of evidence. To fix that issue, reread the text again looking for patterns in word choice and syntax, meaningful figurative language and imagery, literary devices, and other rhetorical choices, looking for additional types of evidence to support your argument.
  • After you include evidence, offer your own interpretation and explain how this evidence proves the point you make in your thesis.
  • Don’t summarize or speak generally about the author and the text. Everything you write must be backed up with evidence.
  • Don’t let quotes speak for themselves. After every piece of evidence you include, make sure to explain your interpretation. Also, connect the evidence to your overarching argument.

Sophistication (0-1 point)

In this case, sophistication isn’t about how many fancy vocabulary words or how many semicolons you use. According to College Board , one point can be awarded to AP Lang Rhetorical Analysis essays that “demonstrate sophistication of thought and/or a complex understanding of the rhetorical situation” in any of these three ways:

  • Explaining the significance or relevance of the writer’s rhetorical choices.
  • Explaining the purpose or function of the passage’s complexities or tensions.
  • Employing a style that is consistently vivid and persuasive.

Note that you don’t have to achieve all three to earn your sophistication point. A good way to think of this rubric category is to consider it a bonus point that you can earn for going above and beyond in depth of analysis or by writing an especially persuasive, clear, and well-structured essay. In order to earn this point, you’ll need to first do a good job with your thesis, evidence, and commentary.

  • Focus on nailing an argumentative thesis and multiple types of evidence. Getting these fundamentals of your essay right will set you up for achieving depth of analysis.
  • Explain how each piece of evidence connects to your thesis.
  • Spend a minute outlining your essay before you begin to ensure your essay flows in a clear and cohesive way.
  • Steer clear of generalizations about the author or text.
  • Don’t include arguments you can’t prove with evidence from the text.
  • Avoid complex sentences and fancy vocabulary words unless you use them often. Long, clunky sentences with imprecisely used words are hard to follow.

AP Lang Rhetorical Analysis: Sample Prompt

The sample prompt below is published online by College Board and is a real example from the 2021 AP Exam. The prompt provides background context, essay instructions, and the text you need to analyze. For sake of space, we’ve included the text as an image you can click to read. After the prompt, we provide a sample high scoring essay and then explain why this AP Lang Rhetorical Analysis essay example works.

Suggested time—40 minutes.

(This question counts as one-third of the total essay section score.)

On February 27, 2013, while in office, former president Barack Obama delivered the following address dedicating the Rosa Parks statue in the National Statuary Hall of the United States Capitol building. Rosa Parks was an African American civil rights activist who was arrested in 1955 for refusing to give up her seat on a segregated bus in Montgomery, Alabama. Read the passage carefully. Write an essay that analyzes the rhetorical choices Obama makes to convey his message.

In your response you should do the following:

  • Respond to the prompt with a thesis that analyzes the writer’s rhetorical choices.
  • Select and use evidence to support your line of reasoning.
  • Explain how the evidence supports your line of reasoning.
  • Demonstrate an understanding of the rhetorical situation.
  • Use appropriate grammar and punctuation in communicating your argument.

AP Lang Rhetorical Analysis Essay Example

In his speech delivered in 2013 at the dedication of Rosa Park’s statue, President Barack Obama acknowledges everything that Parks’ activism made possible in the United States. Telling the story of Parks’ life and achievements, Obama highlights the fact that Parks was a regular person whose actions accomplished enormous change during the civil rights era. Through the use of diction that portrays Parks as quiet and demure, long lists that emphasize the extent of her impacts, and Biblical references, Obama suggests that all of us are capable of achieving greater good, just as Parks did.

Although it might be a surprising way to start to his dedication, Obama begins his speech by telling us who Parks was not: “Rosa Parks held no elected office. She possessed no fortune” he explains in lines 1-2. Later, when he tells the story of the bus driver who threatened to have Parks arrested when she refused to get off the bus, he explains that Parks “simply replied, ‘You may do that’” (lines 22-23). Right away, he establishes that Parks was a regular person who did not hold a seat of power. Her protest on the bus was not part of a larger plan, it was a simple response. By emphasizing that Parks was not powerful, wealthy, or loud spoken, he implies that Parks’ style of activism is an everyday practice that all of us can aspire to.

AP Lang Rhetorical Analysis Essay Example (Continued)

Even though Obama portrays Parks as a demure person whose protest came “simply” and naturally, he shows the importance of her activism through long lists of ripple effects. When Parks challenged her arrest, Obama explains, Martin Luther King, Jr. stood with her and “so did thousands of Montgomery, Alabama commuters” (lines 27-28). They began a boycott that included “teachers and laborers, clergy and domestics, through rain and cold and sweltering heat, day after day, week after week, month after month, walking miles if they had to…” (lines 28-31). In this section of the speech, Obama’s sentences grow longer and he uses lists to show that Parks’ small action impacted and inspired many others to fight for change. Further, listing out how many days, weeks, and months the boycott lasted shows how Parks’ single act of protest sparked a much longer push for change.

To further illustrate Parks’ impact, Obama incorporates Biblical references that emphasize the importance of “that single moment on the bus” (lines 57-58). In lines 33-35, Obama explains that Parks and the other protestors are “driven by a solemn determination to affirm their God-given dignity” and he also compares their victory to the fall the “ancient walls of Jericho” (line 43). By of including these Biblical references, Obama suggests that Parks’ action on the bus did more than correct personal or political wrongs; it also corrected moral and spiritual wrongs. Although Parks had no political power or fortune, she was able to restore a moral balance in our world.

Toward the end of the speech, Obama states that change happens “not mainly through the exploits of the famous and the powerful, but through the countless acts of often anonymous courage and kindness” (lines 78-81). Through carefully chosen diction that portrays her as a quiet, regular person and through lists and Biblical references that highlight the huge impacts of her action, Obama illustrates exactly this point. He wants us to see that, just like Parks, the small and meek can change the world for the better.

AP Lang Rhetorical Analysis Essay Example: Why It Works

We would give the AP Lang Rhetorical Analysis essay above a score of 6 out of 6 because it fully satisfies the essay’s 3 rubric categories: Thesis, Evidence and Commentary, and Sophistication . Let’s break down what this student did:

The thesis of this essay appears in the last line of the first paragraph:

“ Through the use of diction that portrays Parks as quiet and demure, long lists that emphasize the extent of her impacts, and Biblical references, Obama suggests that all of us are capable of achieving greater good, just as Parks did .”

This student’s thesis works because they make a clear argument about Obama’s rhetorical choices. They 1) list the rhetorical choices that will be analyzed in the rest of the essay (the italicized text above) and 2) include an argument someone else might disagree with (the bolded text above).

Evidence and Commentary:

This student includes substantial evidence and commentary. Things they do right, per the AP Lang Rhetorical Analysis rubric:

  • They include lots of specific evidence from the text in the form of quotes.
  • They incorporate 3 different types of evidence (diction, long lists, Biblical references).
  • After including evidence, they offer an interpretation of what the evidence means and explain how the evidence contributes to their overarching argument (aka their thesis).

Sophistication

This essay achieves sophistication according to the AP Lang Rhetorical Analysis essay rubric in a few key ways:

  • This student provides an introduction that flows naturally into the topic their essay will discuss. Before they get to their thesis, they tell us that Obama portrays Parks as a “regular person” setting up their main argument: Obama wants all regular people to aspire to do good in the world just as Rosa Parks did.
  • They organize evidence and commentary in a clear and cohesive way. Each body paragraph focuses on just one type of evidence.
  • They explain how their evidence is significant. In the final sentence of each body paragraph, they draw a connection back to the overarching argument presented in the thesis.
  • All their evidence supports the argument presented in their thesis. There is no extraneous evidence or misleading detail.
  • They consider nuances in the text. Rather than taking the text at face value, they consider what Obama’s rhetorical choices imply and offer their own unique interpretation of those implications.
  • In their final paragraph, they come full circle, reiterate their thesis, and explain what Obama’s rhetorical choices communicate to readers.
  • Their sentences are clear and easy to read. There are no grammar errors or misused words.

AP Lang Rhetorical Analysis Essay—More Resources

Looking for more tips to help your master your AP Lang Rhetorical Analysis Essay? Brush up on 20 Rhetorical Devices High School Students Should Know and read our Tips for Improving Reading Comprehension . If you’re ready to start studying for another part of the AP English Exam, find more expert tips in our How to Write the AP Lang Synthesis blog post.

Considering what other AP classes to take? Read up on the Hardest AP Classes .

  • High School Success

Christina Wood

Christina Wood holds a BA in Literature & Writing from UC San Diego, an MFA in Creative Writing from Washington University in St. Louis, and is currently a Doctoral Candidate in English at the University of Georgia, where she teaches creative writing and first-year composition courses. Christina has published fiction and nonfiction in numerous publications, including The Paris Review , McSweeney’s , Granta , Virginia Quarterly Review , The Sewanee Review , Mississippi Review , and Puerto del Sol , among others. Her story “The Astronaut” won the 2018 Shirley Jackson Award for short fiction and received a “Distinguished Stories” mention in the 2019 Best American Short Stories anthology.

  • 2-Year Colleges
  • ADHD/LD/Autism/Executive Functioning
  • Application Strategies
  • Best Colleges by Major
  • Best Colleges by State
  • Big Picture
  • Career & Personality Assessment
  • College Essay
  • College Search/Knowledge
  • College Success
  • Costs & Financial Aid
  • Data Visualizations
  • Dental School Admissions
  • Extracurricular Activities
  • General Knowledge
  • Graduate School Admissions
  • High Schools
  • Homeschool Resources
  • Law School Admissions
  • Medical School Admissions
  • Navigating the Admissions Process
  • Online Learning
  • Outdoor Adventure
  • Private High School Spotlight
  • Research Programs
  • Summer Program Spotlight
  • Summer Programs
  • Teacher Tools
  • Test Prep Provider Spotlight

“Innovative and invaluable…use this book as your college lifeline.”

— Lynn O'Shaughnessy

Nationally Recognized College Expert

College Planning in Your Inbox

Join our information-packed monthly newsletter.

What are your chances of acceptance?

Calculate for all schools, your chance of acceptance.

Duke University

Your chancing factors

Extracurriculars.

how to write an ap lang rhetorical analysis essay

How to Write the AP Lang Rhetorical Essay

Do you know how to improve your profile for college applications.

See how your profile ranks among thousands of other students using CollegeVine. Calculate your chances at your dream schools and learn what areas you need to improve right now — it only takes 3 minutes and it's 100% free.

Show me what areas I need to improve

What’s Covered:

What is the ap lang rhetorical essay, tips for writing the ap lang rhetorical essay.

  • AP Lang Rhetorical Essay Example

How Will AP Scores Affect College Chances?

The AP English Language Exam is one of the most common AP exams you can take. However, the average score on the exam in 2020 was a 2.96 out of 5. While this may seem a bit low, it is important to note that over 550,000 students take the exam annually. With some preparation and knowing how to study, it is totally possible to do well on this AP exam.

The AP Lang Rhetorical Essay is one section of the AP English Language Exam. The exam itself is 3 hours and 15 minutes long, and is broken into two sections. The first part of the exam is a 60 minute, 45-question multiple-choice section. The questions on this part of the exam will test your ability to read a passage and then interpret its meaning, style, and overall themes. After the multiple-choice section, there is a section lasting 2 hours and 15 minutes with three “free response” essays. This includes the synthesis essay, the rhetorical analysis essay, and the argument essay. 

  • In the synthesis essay , you will have to develop an argument using pieces of evidence provided to you. 
  • The argumentative essay will have you pick a side in a debate and argue for or against it.
  • The rhetorical essay requires that you discuss how an author’s written passage contributes to a greater meaning or theme. 

The rhetorical essay is perhaps the most unique of all AP Lang exam essays because it requires the test taker to analyze and interpret the deeper meanings of the passage and connect them to the author’s writing style and writing syntax in only 40 minutes. This essay can be the trickiest because it requires you to have knowledge of rhetorical strategies and then apply them to a passage you’ve never seen before.

1. Outline Your Essay Before Writing

One of the most important parts of the AP Lang essays is structuring your essay so that it makes sense to the reader. This is just as important as having good content. For this essay in particular, you’ll want to read the passage first and write a brief outline of your points before you begin the essay. This is because you will want to write the essay using the passage chronologically, which will be discussed in detail below.

2. Understand Rhetorical Strategies 

If you feel like you don’t know where to start as you prepare to study for the rhetorical essay portion of the exam, you aren’t alone. It is imperative that you have a grasp on what rhetorical strategies are and how you can use them in your essay. One definition of rhetoric is “language carefully chosen and arranged for maximum effect.” This can include types of figurative language (metaphor, simile, personification, pun, irony, etc.) elements of syntax (parallelism, juxtaposition, anthesis, anaphora, etc), logical fallacies, or persuasive appeals. Overall, there are many elements that you can analyze in an essay and having a good grasp on them through practice and memorization is important.

3. Keep the Essay Well Structured 

Even if you understand the various rhetorical strategies you can use, where do you begin? First of all, you’ll want to write a strong introduction that outlines the purpose of the piece. At the end of this introduction, you will write a thesis statement that encapsulates all the rhetorical strategies you discuss. Perhaps these are style elements, tone, or syntax. Be sure to be specific as you list these.

Next, you will create your body paragraphs. As you discuss the rhetorical elements in the piece and tie them back to the work’s meanings, be sure to discuss the points in chronological order. You don’t have to discuss every single strategy, but just pick the ones that are most important. Be sure to cite the line where you found the example. At the end of the essay, write a short conclusion that summarizes the major points above.

4. Be Sure to Explain Your Examples

As you write the essay, don’t just list out your examples and say something like “this is an example of ethos, logos, pathos.” Instead, analyze how the example shows that rhetoric device and how it helps the author further their argument. As you write the rhetorical essay, you’ll want to be as specific and detail-focused as possible. 

how to write an ap lang rhetorical analysis essay

Discover your chances at hundreds of schools

Our free chancing engine takes into account your history, background, test scores, and extracurricular activities to show you your real chances of admission—and how to improve them.

AP Lang Rhetorical Analysis Essay Example

Below is a prompt and example for a rhetorical essay, along with its score and what the writer did well and could have improved:

The passage below is an excerpt from “On the Want of Money,” an essay written by nineteenth-century author William Hazlitt. Read the passage carefully. Then write an essay in which you analyze the rhetorical strategies Hazlitt uses to develop his position about money.

how to write an ap lang rhetorical analysis essay

Student essay example:

In his essay, Hazlitt develops his position on money through careful use of adjectives and verbs, hypothetical situations, and images. His examples serve to impress upon the reader the highly negative consequences of being in “want of money.”

Hazlitt’s word choice in his opening phrase provides an example of his technique in the rest of the essay. It is not necessary to follow “literally” with “truly” yet his repetition of the same ideas emphasizes his point. In his next sentence, one that lasts forty-six lines, Hazlitt condignly repeats similar ideas, beating into his audience the necessity of having money in this world. The parallelism throughout that one long sentence, “it is not to be sent for to court, or asked out to dinner…it is not to have your own opinion consulted or sees rejected with contempt..” ties the many different situations Haziltt gives together. What could have become a tedious spiel instead becomes a melodious recitation, each example reminding you of one before it, either because of the similarities in structure or content. Hazlitt addresses many different negative effects of not having money but manages to tie them together with his rhetorical strategies. 

The diction of the passage fully relays Hazlitt’s position about money. In every example he gives a negative situation but in most emphasizes the terrible circumstance with strong negative adjectives or verbs. “Rejected,” “contempt,” “disparaged,” “scrutinized,” “irksome,” “deprived,” “assailed” “chagrin;” the endless repetition of such discouragement shows how empathetically Hazlitt believes money is a requisite for a happy life. Even the irony of the last sentences is negative, conveying the utter hopelessness of one without money. Through one may have none in life, pitiless men will proceed to mock one’s circumstances, “at a considerable expense” after death! 

In having as the body of his essay one long sentence, Hazlitt creates a flow that speeds the passage along, hardly giving the reader time to absorb one idea before another is thrown at him. The unceasing flow is synonymous with Hazlitt’s view of the life of a person without money: he will be “jostled” through life, unable to stop and appreciate the beauty around him or to take time for his own leisure. 

The score on this essay was a 6 out of 6. This essay started out very strong as the student had a concrete thesis statement explaining the strategies that Hazlitt used to develop his position on money as well as Hazlitt’s belief on the topic. In the thesis statement, the student points out that adjectives, verbs, hypothetical situations, and images help prove Hazlitt’s point that wanting money can be problematic. 

Next, the student broke down their points into three main subsections related to their thesis. More specifically, the student first discusses word choice of repetition and parallelism. When the student discusses these strategies, they list evidence in the paragraph that can be found chronologically in Hazlitt’s essay. The next paragraph is about diction, and the student used specific adjectives and verbs that support this idea. In the last paragraph, the student emphasized how the speed and flow of the essay helped describe Hazlitt’s viewpoint on life. This last concluding sentence is particularly thoughtful, as it goes beyond the explicit points made in the essay and discusses the style and tone of the writing. 

It is important to remember that in some ways, the rhetorical essay is also an argumentative essay, as the student must prove how certain rhetorical strategies are used and their significance in the essay. The student even discussed the irony of the paragraph, which is not explicit in the passage.

Overall, this student did an excellent job organizing and structuring the essay and did a nice job using evidence to prove their points. 

Now that you’ve learned about the AP Lang rhetorical essay, you may be wondering how your AP scores impact your chances of admission. In fact, your AP scores have relatively little impact on your admissions decision , and your course rigor has much more weight in the application process.

If you’d like to know your chances of admission, be sure to check out our chancing calculator! This tool takes into account your classes, extracurriculars, demographic information, and test scores to understand your chances at admission at over 600 schools. Best of all, it is completely free!

how to write an ap lang rhetorical analysis essay

Related CollegeVine Blog Posts

how to write an ap lang rhetorical analysis essay

AP English Language and Composition: Sample Rhetorical Analysis and Synthesis Questions

April 9, 2024.

AP English Language Sample Rhetorical Analysis and Synthesis Questions

The Rhetorical Analysis and Synthesis Essays are two of the three essays you’ll need to write as part of the AP English Language and Composition Exam . Read on for a sample of each, as well as tips for how to answer them. 

AP English Language and Composition: Sample Rhetorical Analysis Question

Read the following passage published back in 1967 by The New York Times. Then write an essay in which you analyze how the structure of the passage and the use of language help convey the writer’s views.

Sample Question Instructions:

  • Respond to the prompt with a thesis that may establish a line of reasoning.
  • Select and use evidence to develop and support the line of reasoning.
  • Explain the relationship between the evidence and the thesis.
  • Demonstrate an understanding of the rhetorical situation.
  • Use appropriate grammar and punctuation in communicating the argument.

Americans and Western Europeans, in their sensitivity to lingering problems around them, tend to make science and progress their scapegoats. There is a belief that progress has precipitated widespread unhappiness, anxieties, and other social and emotional problems. Science is viewed as a cold mechanical discipline having nothing to do with human warmth and the human spirit. 

But to many of us from the nonscientific East, science does not have such repugnant associations. We are not afraid of it, nor are we disappointed by it. We know all too painfully that our social and emotional problems festered long before the age of technology. To us, science is warm and reassuring. It promises hope. It is helping us at long last gain some control over our persecutory environments, alleviating age-old problems—not only physical but also, and especially, problems of the spirit.

Shiraz, for example, a city in southern Iran, has long been renowned for its rose gardens and nightingales; its poets, Sadi and Hafiz; and its mystical, ascetic philosophy, Sufism. Much poetry has been written in glorification of the spiritual attributes of this oasis city. And to be sure, Shiraz is a green, picturesque town, with a quaint bazaar and refreshing gardens. But in this “romantic” city thousands of emotionally disturbed and mentally retarded men, women, and children were, until recently, kept in chains in stifling prison cells and lunatic asylums. 

Every now and again, some were dragged, screaming and pleading, to a courtyard and flogged for not behaving “normally.” But for the most part, they were made to sit against damp walls, their hands and feet locked in chains, and thus immobilized, without even a modicum of affection from their helpless families and friends, they sat for weeks and months and years—often all their lives. Pictures of these wretched men, women, and children can still be seen in this “city of poetry,” this “city with a spiritual way of life.” 

It was only recently that a wealthy young Shirazi who, against the admonitions of his family, had studied psychology at the University of Tehran and foreign universities, returned to Shiraz and after considerable struggle with city officials succeeded in opening a psychiatric clinic, the first in those regions. After still more struggle, he arranged to have the emotionally disturbed and the mentally retarded transferred from prison to their homes, to hospitals, and to his clinic, where he and his staff now attend them. 

They are fortunate. All over Asia and other backward areas, emotionally disturbed men and women are still incarcerated in these medieval dungeons called lunatic asylums. The cruel rejection and punishment are intended to teach them a lesson or help exorcise evil spirits. 

The West, still bogged down in its ridiculous romanticism, would like to believe that emotional disturbances, dope addiction, delinquency are all modern problems brought on by technological progress, and that backward societies are too spiritual and beautiful to need the ministrations of science. But while the West can perhaps afford to think this way, the people of backward lands cannot. . . . 

. . .The obstacles are awesome, the inertia too entrenched, the people’s suffering too anguished, their impatience too eruptive. Moreover, the total cultural reorganizations such as Asia and Africa are undergoing inevitably engender their own temporary dislocations and confusions. But their goals, the direction, remain constant. We are on the move, however awkwardly at first, to a saner, better world.

How to Answer the AP English Language and Composition Rhetorical Analysis Question

Go back to the original question, which asks you to analyze two features of the passage: (1) its structure, or organization, and (2) its language. The first aspect is fairly specific. As you read the passage, you need to observe what the author discusses first, second, third, and so on. Your essay should explain not only the order of ideas but the reasons the author may have chosen that order. 

The second part of the question is more general. It invites you to analyze the use of language, which may include the author’s choice of words (diction), syntax (word order), figures of speech, use of evidence (such as statistics or logical reasoning), sentence structure, rhythm, sound, tone, or just about any other characteristics of style and rhetoric you choose. 

Although the question directs you to write about two different aspects of the passage, the essay itself should be unified. That is, a good essay should not consist of, say, two disparate paragraphs, one exclusively devoted to structure and another to language. Rather, the essay should include material that shows the interrelationship of structure and language in the passage and how those elements contribute to the meaning and effect of the passage. This might be covered in a separate paragraph, or it could be woven into the overall fabric of the essay. 

Before you begin to write, read the passage at least twice: once for an overview and once as you write your analysis. You may notice early on that the opening paragraph contains generalizations about Westerners’ concepts of science and progress. Then the author contrasts the Western view of science and progress with the Eastern view. Immediately, you see that the author, by using the first-person pronoun (as in “many of us”) is speaking from the perspective of an Easterner. Consequently, his discussion of Eastern views is apt to come across as more well-informed, more authoritative, perhaps more personal. 

To support his position, the author gives an extended example—the city of Shiraz—to illustrate just how different the East is from the West. The description and vivid images of Shiraz memorably convey the idea that the “spiritual way of life” has a side to it that many Westerners don’t know about. This is the heart of the passage. The use of quotation marks around “romantic” and “city of poetry” is meant to point out the discrepancy between the idealized and real versions of Shiraz. 

Nearing the end, the author reiterates his initial contrast between West and East, with emphasis on the East. The last paragraph offers a generalized statement about conditions in Asia and Africa, reminding the reader of the contrast made at the very beginning of the passage. Tying the end to the beginning of the passage creates a sense of unity—a desirable feature in any piece of writing.

AP English Language and Composition: Sample Argument Question

The following paragraph is adapted from Mirror for Man, a book written by anthropologist Clyde Kluckhorn in the middle of the twentieth century. Read the passage carefully. Then, write an essay that examines the extent to which the author’s characterization of the United States holds true today. Use appropriate evidence to support your argument. 

Sample Question Instructions: 

  • Respond to the prompt with a thesis that may establish a line of reasoning. 
  • Select and use evidence to develop and support the line of reasoning. 
  • Explain the relationship between the evidence and the thesis. 
  • Demonstrate an understanding of the rhetorical situation. 

Technology is valued as the very basis of the capitalistic system. Possession of gadgets is esteemed as a mark of success to the extent that persons are judged not by the integrity of their characters or by the originality of their minds but by what they seem to be—so far as can be measured by their wealth or by the variety and material goods which they display. “Success” is measured by their investments, homes, and lifestyles— not by their number of mistresses as in some cultures.

How to Answer the AP English Language and Composition Argument Question

Whether you agree, disagree, or have mixed views on the content of the passage, your job is to write a convincing argument that expresses your opinion. Initially, the word argument may suggest conflict or confrontation. But rest assured that your essay need not be combative. Rather, make it a calmly-reasoned explanation of your opinion on a debatable subject. Your goal is to persuade the reader that your opinion, supported by examples, facts, and other appropriate evidence, is correct. 

If you have strong feelings about the topic, of course you should state them in your essay. But express them in calm, rational language. Be mindful that the essay should not be an emotional rant for or against the issue. 

Consider first whether you agree with Kluckhorn’s definition of “success.” Is it, as Kluckhorn asserts, measured by income and material possessions? Or do you think that a more accurate standard of success in today’s America should be determined by less tangible criteria—things such as happiness or self-respect? Or do you stand somewhere in between those two extremes? 

The actual position you take on the issue is less crucial than your ability to support it fully by drawing from your knowledge, background, experience, or observation. Regardless of your position, be sure to include more than one example. An argument that relies on a single example, however compelling, will fall flat. 

In the prompt, Kluckhorn’s notion of success seems to refer broadly to American society. Resist responding in kind. That is, a short essay shouldn’t focus on the whole of society but only on an identifiable segment—perhaps college-educated professionals or urban, blue- collar Americans. The point is that a narrowly focused essay on a limited topic will always turn out better than one that tries to cover too much ground in just a few paragraphs.

AP Biology Resources

  • About the AP Biology Exam
  • Top AP Biology Exam Strategies
  • Top 5 Study Topics and Tips for the AP Biology Exam
  • AP Biology Short Free-Response Questions
  • AP Biology Long Free-Response Questions

AP Psychology Resources

  • What’s Tested on the AP Psychology Exam?
  • Top 5 Study Tips for the AP Psychology Exam
  • AP Psychology Key Terms
  • Top AP Psychology Exam Multiple-Choice Question Tips
  • Top AP Psychology Exam Free Response Questions Tips
  • AP Psychology Sample Free Response Question

AP English Language and Composition Resources

  • What’s Tested on the AP English Language and Composition Exam?
  • Top 5 Tips for the AP English Language and Composition Exam
  • Top Reading Techniques for the AP English Language and Composition Exam
  • How to Answer the AP English Language and Composition Essay Questions 
  • AP English Language and Composition Exam Sample Essay Questions
  • AP English Language and Composition Exam Multiple-Choice Questions

AP Human Geography Resources

  • What’s Tested On the AP Human Geography Exam?
  • AP Human Geography FAQs
  • AP Human Geography Question Types and Strategies
  • Top 5 Study Tips for the AP Human Geography Exam

FOLLOW ALONG ON SOCIAL

Get in touch with us

Are you sure you want to logout?

Study abroad.

bannerAd

AP Lang Rhetorical Analysis Essay

AP language exams are held for numerous subjects. Students have to choose a specific subject and get higher scores. The higher the score, the higher the chance of pursuing the best 700 colleges or universities overall in the world, including the United States and Canada.

When it comes to AP English language, it involves a section called rhetorical analysis essay. This is a part of three free-response essays that have to be answered within 2 hours and 15 minutes from the overall 3 hours 15 minutes exam. 

parallel

If you are taking the AP Lang exam this year, guidance on how to answer this part will be useful. This article is specially curated to help you score the best. Read on to learn more about the AP language rhetorical analysis essay and get an idea of how to prepare for the associated exam successfully.

What is the AP Lang Rhetorical Analysis Essay ?

AP elaborated that the Advanced Placement is the exam conducted by the College Board in the United States of America. The exam is generally offered at the high school level and helps students to pursue higher education at the university level. The exam holds two sections: MCQs and Rhetorical Essay. MCQ sections are for 1 hour, and the rest of the time is for the essay section. The free-response essay holds three essays: rhetorical analysis essay, synthesis essay, and argumentative essay. 

parallel

  • The rhetorical analysis essay in the AP Lang exam involves students having to discuss how the authors’ contribution to the passage gives a theme or meaning. 
  • A synthesis essay involves students creating arguments on the passage or piece of information delivered to them.
  • An argumentative essay requires students to pick a side ‘for or against ‘ for an argument or debate.

Since we are here to discuss rhetorical analysis essays in AP language, you must know the essay is added to test students’ ability to analyze and interpret the deeper meaning in the provided passage. Through rhetorical essays, the examiner examines how students connect with the author’s style of writing and syntax within 40 minutes. Some students may face challenges while dealing with this essay because it requires a better understanding of rhetorical strategies and the method to apply them.

Tips to Write a Rhetorical Analysis Essay AP Lang ?

Since a rhetorical essay is quite tricky in comparison to other essays, it requires certain tips for a better approach to answering. Let us explore the method of writing a rhetorical essay for the AP Lang exam that helps you score well.

parallel

  • Outline Essay Prior to Writing

Not only good content but providing a readable structure is an important part of the rhetorical analysis essay AP Lang . First, you must read the passage thoroughly and develop a brief outline or key points before writing the essay. This helps you write with respect to the chronology of the given passage and maintain the flow of writing.

  • Understand Rhetorical Strategies

You aren’t alone in thinking about where to start writing a rhetorical essay. Many students face the same and are required to learn and implement rhetorical strategies. First, understand what rhetoric actually means. It refers to language sensibly chosen and structured for an impressive effect on the audience. This involves persuasive appeal, logical fallacies, and syntax such as anaphora, anthesis, parallelism, and so on. There is a vast range of elements that you can assess in the provided essay and develop a strong grasp with consistent practice.

parallel

  • Make your Essay Well-Structured

Sometimes, students understand the rhetorical strategies but still get confused about where to initiate. It is recommended to start with an introduction that delivers the purpose of your writing. In the last introductory line, you must talk about the rhetorical strategies you will discuss in the piece. However, there are so many styles, syntax, and tones. You must be specific while listing them and then move to develop a body paragraph. 

Now, you have to collect all the rhetorical strategies you mentioned in the introduction to discuss your point of view chronologically. Be specific while discussing strategy, as only the crucial ones among them must be discussed. Never forget to cite the line from the original passage. Your write-up must maintain the flow and should include relativity among the paragraphs. End the rhetorical essay by summarising key points.

parallel

  • Never Forget to Explain your Examples

Do not just state the examples or deliver statements like ‘this is an example of pathos or logos.’ It is advisable to explain the example you have listed in context to the rhetorical elements you have mentioned and how it aids the author in their viewpoint. Stay detailed yet precise while writing the rhetorical analysis essay AP Lang .

AP Lang Rhetorical Analysis Essay Rubric

Did you know that the rhetorical analysis essay AP Lang is graded into three rubric categories? Students must pay heed to the categories, as the examiner seeks specific things in each of them. Also, learn about some dos and don’ts to score well.

parallel

When it comes to grading the thesis of the rhetorical analysis essay AP Lang , there is nothing nebulous. Either you get one point, or you lose one. Thesis points help you get close to higher scores, and thus, you must be mindful of the following points.

parallel

  • Developing an argument states that your interpretation leads to a risk of disagreement. Thus, your thesis statements must be in context with the author’s rhetorical choice. 
  • If you create a phrase in your mind initiating with ‘I think that..’, your phrase should not go in a negative argumentative direction.
  • Never provide a thesis with summaries but not an argument.
  • Try not to provide the thesis with repeated prompts.

Evidence is provided to prove the argumentative context in the thesis. This rubric category is graded from 0 to 4 points. To score higher, you must follow the points below.

  • You must aim to provide multiple types of evidence in your argumentative thesis.
  • Each statement you provide must be backed up with evidence in context to the text or arguments about the author’s theoretical choices.
  • Add more and more evidence, which must be specific.
  • Examine whether your evidence is linked with your overarching argument.
  • Deliver your interpretation and never rely on just quotes or phrases. 
  • Steer clear of generalization for text or author
  • Avoid quotes that speak for themselves. You must elaborate on the evidence you provided.
  • Sophistication

According to The College Board, this category holds 0 to 1 points. The higher grading in rhetorical analysis essays indicates the delivery of the sophistication of thoughts or a complex understanding of rhetorical elements. In this, you have to mainly focus on the number of semicolons you use and not the fancy terms. Here is what you need to follow:

  • Focus on delivering the right connection between the thesis and your evidence.
  • Create a brief framework and then proceed with writing the essay in the right flow to stay precise and clear in your piece.
  • Do not include arguments that you won’t be able to provide evidence for.
  • Ignore complex or fancy words or phrases that are hard to follow.

Ways to Improve Your Rhetorical Analysis Essay AP Lang

Now that you know what a rhetorical analysis essay is and how it is developed, you must be thinking about how tricky it is to understand the passage. Many find themselves in a completely blank position when it comes to where to begin during the exam. They find the rhetorical analysis essay AP Lang quite challenging to deal with and understand the author’s perspective and viewpoint. Practicing rhetorical strategies does not help if you do not have the right approach, which only comes from the field experts and their guidance.  Turito offers a platform where you get mentors to teach you all the concepts and approaches you apply in writing effective rhetorical essays in less time. Their course helps you get higher scores, which you can use to get college admission to your favorite college and country. Hey, you ambitious one, what are you waiting for? Contact us now and give your dreams a flight!

Frequently Asked Questions

What courses do turito offer.

Turito offers several courses to help you prepare. Here are some of them for your reference. 1. Foundation Course 2. IIT JEE 3. NEET 4. Study Abroad 5. Universal Program 6. PSAT 7. AP 8. IELTS

Can I ask queries on Turito?

Yes absolutely! Turito offers a platform where you can ask your concerns and queries. You will get answers from the field expert that can let your worries fade away.

Is Turito genuine?

Turito is students’ most trusted learning platform, among others. Turito believes in transparency and thus provides genuine feedback from students and parents on the official website. At Turito, we offer guaranteed success for hardworking students. Thus, there isn’t any negative feedback to hide. If you have any queries, you can write to us at [email protected]. Our experts will connect and assist you in no time.

AP Lang Rhetorical Analysis Essay

Relevant Articles

AP Lang Synthesis Essay

Steps to Draft AP Lang Synthesis Essay

The synthesis essay AP lang is part of the AP …

Steps to Draft AP Lang Synthesis Essay Read More »

AP Calculus AB Exam

Strategies for Success: A Complete Overview of the AP Calculus AB Exam

The AP Calculus AB Exam serves as an important element …

Strategies for Success: A Complete Overview of the AP Calculus AB Exam Read More »

Digital AP Exams

Preparing for Success: A Guide to the Features of Digital AP Exams

Digital  Advanced Placement (AP) exams bring about a huge change …

Preparing for Success: A Guide to the Features of Digital AP Exams Read More »

card img

With Turito Study Abroad

card img

Get an Expert Advice from Turito

card img

With Turito CAP.

card img

With Turito Coding.

card img

With Turito RoboNinja

card img

1-on-1 tutoring for the undivided attention

The Practical English Teacher

Free Resources for Secondary English Teachers

  • Aug 23, 2023

Teaching the AP Language and Composition Rhetorical Analysis Essay

Updated: Nov 26, 2023

Every year, my AP Language and Composition students struggle with the rhetorical analysis essay. Over the years, I've come up with various graphic organizers to walk them through the process, and I also show them various models, but this only gets students so far.  I teach highly motivated students, and they want to know  exactly  what they need to do to earn a high score.

This year,  after i graded a stack of so-so papers, i realized that i was going to have 60 students chasing me down and demanding to know how to get a better grade. i suffer from fear of student bombardment (fosb), so i try and anticipate what students are going to be concerned about and address it in class. below are the materials i use to teach the rhetorical analysis essay..

AP Lang & Comp Bundle

I had already used all of my rhetorical analysis graphic organizers and some of my AP models. My students were starting to understand the rhetorical analysis essay but not quite. They were identifying devices and trying to connect to effect, but they weren't keep the big picture in mind.  They were also picking out small, unimportant devices to analyze, or they were skipping giants chunks of the passage. I realized that in order to show them exactly what I wanted,  I was going to have to model the entire process for them. Tedious? Yes. But I knew it was the only way I would be able to show them exactly what I wanted, and therefore prevent the student onslaught after class. (WHY DIDN'T I GET A BETTER GRADE!?!?)

I did this lesson after students attempted to analyze the 2005 form b maria w. stewart prompt, but you can do it after any rhetorical analysis attempt. the reason i do it after they write the essay is because they are not invested in the process until they get their grades back. that's when the realize that they probably should stop ignoring me and pay attention. the lesson went well and i didn't have any students chasing me down afterwards. you will need about ninety minutes. you can use the steps below with your own passage, or you can use this ready made "intro the rhetorical analysis essay" for sale on my tpt page. i use sojourner truth's "ain't i a woman".

Introduction to the Rhetorical Analysis

Here is what to do:

1. Give students back their essays and briefly discuss what they did well, but acknowledge that there's work to be done.

2. Give students a handout titled "How to Write a Rhetorical Analysis Essay." This is just a clean document that lists the steps students should follow when attempting to write a rhetorical analysis essay.  I sell it on TpT, but it is not necessary for this lesson:  https://www.teacherspayteachers.com/Product/How-to-Write-a-Rhetorical-Analysis-Essay-for-the-AP-Lang-and-Comp-Exam-1541269 

How to Write the Rhetorical Analysis Essay

3. Choose a short passage to work with or use an old AP prompt. Show students how to annotate the first paragraph (summary/function/devices), and then have them annotate the second paragraph on their own and share out.

4. Next go over how to write a rhetorical analysis thesis statement. I have my students write a SOAPStone thesis. They use it in their AP history classes and it works well for the rhetorical analysis essay, so I figure... the fewer acronyms to remember the better. The "SOAPStone" letter meanings are as follows: S=Subject, O=Occassion, A=Audience, P=Purpose, S=Style, and "tone"=tone. Students should identify all of this information about the text they are analyzing and then tie it together in one or two sentences at the end of their introductory paragraphs. Kids always ask if they should list literary devices but I am partial to them identifying and discussing the devices in the body. Really, I don't care it's up to them.

5. Show students how to write a full intro paragraph by typing it out on the spot and thinking aloud. Make students take notes by hand while you are typing.

6. Show students what the body paragraph of a rhetorical analysis essay to look like. I literally type out the paragraph and think aloud while I am writing it. I also make them copy this down by hand.

6. Repeat for another paragraph.

7. Ask students to write their body paragraph own. Despite having a model paragraph right in front of them, students typically make the same three mistakes on their first attempt at the rhetorical analysis:

They do not put a device in their assertion

They speak in generalities in their commentary

They do not include exact evidence.

As students are working, I try and circulate and catch kids who are making one of the above mistakes. Students submit their attempts at the end of class.

During Planning/Before Next Class

8. Look through the paragraphs students submitted and divide them up into “strong,” “medium” and “weak.” Then make groups of three with one student from each category. Basically, you want to match the students who DO understand the rhetorical analysis essay to be paired up with students who DO NOT understand the rhetorical analysis essay. The stronger students can explain and model how to write the essay to the kids who did not get it the first time around.

9. Review any major misunderstandings that you saw in their paragraphs.

10. Start the process over again. Give students Dave Barry’s “Turkeys in the Kitchen.” (complete lesson with handouts for sale here ), and ask them to take out their "How to Write a Rhetorical Analysis Essay" handout . Go th rough the steps again. Students read the prompt and the passage on their own and annotate. We share out some annotations, and then we go over how to write the thesis statement as a class.

11. Put students into the mixed-ability groups that you made the day before. The students divide up the into “beginning” “middle” and “end” and each person is responsible for writing 1 body paragraph that will be part of a whole essay written by the group. I have students write their paragraph on a Google doc and then share it with me so that I can check in on them as they are writing. I use the commenty function to leave feedback, but if a student is really confused, I call them up to my desk for a quick conference.

12. Once all students in the group have written their paragraph, they need to read the paragraphs of the other two members in the group and give one piece of feedback.

13. Submit essay to teacher.

14. Review all the steps with the students one more time and then have them do a their first timed rhetorical analysis. Give them 55 minutes to complete it, then I look over the essays and figure out my next lesson.

No first attempt at a rhetorical analysis is pretty. The essay is just too new and unfamiliar to my students. I’ve just learned to accept this stumble as an inevitable part on the path to mastery. My students hate it for this reason, but they feel pretty good once they get the hang of it.

Rhetorical Analysis Essay Materials

Rhetorical Analysis Introductory Lesson (w. Sojourner Truth) (paid product)

Introduction to the Rhetorical Analysis Essay

How to Write a Rhetorical Analysis for the AP Exam (paid product)

How to Write the Rhetorical Analysis Essay

Rhetorical Analysis Essay Template (paid product)

Rhetorical Analysis Step by Step

Rhetorical Analysis Thesis Statements

3 Graphic Organizers for the Rhetorical Analysis Essay (paid product)

Scaffolding for the Rhetorical Analysis Essay

Rhetorical Analysis Group Essay (paid product)

Rhetorical Analysis Group Essay

How to do a Compare/Contrast Rh etorical Analysis PowerPoint

Woolf Rhetorical Analysis Compare/Contrast Prompt

https://amzn.to/3RgEY0Y

  • AP Language & Composition

Recent Posts

How to Start AP Language & Composition

The Language of Composition-2nd Edition

Grizzly Man Clips That Can Be Used in the Classroom

Global Health and Education

2020 ap english exam: how to write a rhetorical analysis essay.

In light of The College Board recently announcing that this year’s AP English Language and Composition exam will be only one question , a 45-minute rhetorical analysis essay,  it would probably be a good idea to freshen up on your essay-writing skills. The benefits of doing this won’t stop once high-school ends- knowing how to write rhetorical analysis essays will also be extremely helpful for future college and professional endeavors where you might be required to examine a text or prepare a report on one. In this article you will learn the breakdown of every section of the rhetorical analysis essay, and what you should be including in order to earn a five this May.

2020 AP English Language and Composition Exam Changes: How to write a 45-minute rhetorical analysis essay?

Introduction.

Before you start writing, make sure to thoroughly read through your text. Try to take note of the SPACECAT features: s peaker, p urpose, a udience, c ontext, e xigence, c hoices, a ppeals, and t one.

After you’ve finished contextualizing the piece itself, you will need to include pertinent information regarding the author, if you are aware of any. What has happened or is happening during their life that caused them to write this piece? If these events did not cause them to write this piece, how did it affect their argument? Are there any personal or political philosophies that they are subscribed to? How does the author themself affect the essay?

The last portion of the first paragraph is the thesis. The thesis emphasizes the purpose of the writing- the why . Why is the author doing any of this? What’s the reason? Your thesis is basically the entire argument of your essay summed up into one sentence. Another thing that’s important to add in your thesis is the effectiveness of the author’s execution of their argument. Did they do a good job of getting their point across? Make sure to have evidence to back this point up later.

Body Paragraphs

Another important aspect to look at is if the author used any appeals in their essay. The appeals are; ethos, appealing by establishing credibility; pathos, appealing to emotion; logos, appealing to logic; kairos, appealing to timeliness. Appeals are great characteristics of any argument and can be very powerful rhetorical choices to discuss in your essay.

Contradiction

The contradiction paragraph is typically the last paragraph before your conclusion. In this section you will address the flaws in the author’s essay as well as in their argument. Be careful- do not share whether or not you agree with the author. This is not something that’s important in a rhetorical analysis essay. You are simply analyzing the techniques used by the author to get their point across- you are not making an argument about whether or not their point was right. Be sure to focus on the weaker parts of your author’s argument, find the holes in it. Try to focus on one or two things that stand out despite all of the rhetorical devices used by them. Adding a contradiction paragraph will show complexity in your essay and give depth to your writing. 

After you’ve summed up the main points made in the text , you should draw some major relevant conclusions from your essay. Talk about the bigger picture. If you had to sum up your entire essay in only three sentences, what would they be? Try to keep this in mind as you approach this part of your writing.

Works Cited

“Examples of Rhetorical Devices.” Your Dictionary. Accessed April 21, 2020. https://examples.yourdictionary.com/examples-of-rhetorical-devices.html .

© Copyright 2020 Moosmosis – All rights reserved

All rights reserved. This essay or any portion thereof may not be reproduced or used in any manner whatsoever without the express written permission of the publisher. 

Please Like our Facebook page to support! 🙂

Share this:, published by elle ward.

Hi! I'm an aspiring journalist and writer. A picture may be worth a thousand words, but a novel can create a thousand pictures. View all posts by Elle Ward

7 replies »

Very thorough and excellent article, Elle! Helpful for many AP English literature and language students.

Like Liked by 1 person

Leave a comment Cancel reply

Thank you for visiting, today’s top posts, our community, choose a language.

Please Like to Support!

All Subjects

Rhetorical Analysis Essay How-To

7 min read • june 18, 2024

Kathryn Howard

Kathryn Howard

Brandon Wu

What is the Rhetorical Analysis Essay?

The is a free-response question in which you analyze any rhetorical strategies that the author uses in the given passage and evaluate/discuss them in a well-formatted essay. 

According to the College Board:

 “The rhetorical analysis free-response essay question presents students with a passage of nonfiction prose of approximately 600 to 800 words. Students are asked to write an essay that analyzes the writer’s rhetorical choices . This question assesses students’ ability to do the following:

  • Respond to the prompt with a thesis that analyzes the writer’s rhetorical choices.
  • Select and use evidence to support your line of reasoning .
  • Explain how the evidence supports your line of reasoning.
  • Demonstrate an understanding of the rhetorical situation .
  • Use appropriate grammar and punctuation in communicating your argument." Essentially, you are being asked to analyze someone’s writing and what strategies they used to help them achieve their purpose.

Rhetorical Analysis Rubric for Scoring

The rhetorical analysis frq is out of 6 points.

✍ The Thesis of your rhetorical analysis essay for AP Lang can score you 1 point right from the start!
* Lacks a The thesis provides summary with no claim*They have a thesis, but it is not in response to the given prompt Student has a justifiable thesis that answers the prompt that analyzes the rhetorical strategies.

how to write an ap lang rhetorical analysis essay

To get this point you need to clearly write a defensible thesis about  the rhetorical choices the author makes. Do not take a stance on the argument the author is making if he/she is making one. You are only talking about rhetorical strategies. 

🎥 Watch: AP Language -   How to Find Rhetorical Strategies

Evidence and Commentary (4 points) 📂

 Only restates thesis, has a lot of repetition, and the information doesn't have to do with the prompt Provides evidence that is mostly general.  : Summarizes evidence but doesn't explain how it applies  Provides some pertinent evidence.  Explains how some of the evidence provided supports the student’s thesis, but does not have good reasoning : Provides specific evidence to support all assertions made Explains how some of the evidence supports a line of reasoning. AND Explains how at least one rhetorical choice in the excerpt helps to achieve the author’s purpose   Provides specific evidence to support all claims in a line of reasoning.  Consistently explains how the evidence supports a line of reasoning. AND Explains how multiple rhetorical strategies help to achieve the author’s purpose.

To get the four points you need to  not only present evidence but  explain why  it supports your thesis and how it contributes to the author’s message.

Sophistication (1 point) 👓

 Does not meet all the standards for one point. Shows sophistication of thought and/or a of the rhetorical situation. 

To get to this point you have to demonstrate a  complex understanding of  both what that purpose was, and how the rhetorical analysis devices aided the author’s purpose.

There are a few ways that you can earn the sophistication point :

  • Explaining the significance or relevance of the writer’s rhetorical choices (given the rhetorical situation).
  • Explaining a purpose or function of the passage’s complexities or tensions.
  • Employing a style that is consistently vivid and persuasive.

You have 40 minutes to complete the rhetorical analysis essay for AP Lang:

  • 12 minutes: Read the text and plan out your essay. (TOBI)
  • 6 minutes: Write your introduction paragraph.
  • 18 minutes: Write 2-3 body paragraphs.
  • 2 minutes: Write a quick conclusion.
  • 2 minutes: Proofread and revise your essay. 🎥 Watch: AP Language -   Rhetorical Analysis Organization and Timing

how to write an ap lang rhetorical analysis essay

How to Maximize Your Time⌚

Outline your rhetorical analysis essay before writing! A great tool for this is a TOBI:

TOBI stands for  thesis *,*  outline *,* and  big idea.

TOBI Outline

  • BI -Big Idea Here is an example of how to use TOBI given a rhetorical analysis prompt:

how to write an ap lang rhetorical analysis essay

T: Luce uses many rhetorical strategies including  pathos *,*  antithesis , and a  humorous tone to soften up her audience before introducing her true reasons for being there. 

- Pathos Appeal

(“There is no audience more forgiving”)

-Antithesis

(“I am happy, I am less happy”)

- Humorous tone

(“consequently, no audience is more forgiving, I hope”)

BI: Today, just like for Luce, it is very difficult to give criticism to your peers.

Note: It is a good idea to make the TOBI about the size of your hand to make sure you don’t spend too much of your precious essay writing time on it.

What if I can't find any rhetorical devices that I recognize?

You can always go back and rely on tone as every piece of literature has one, even if it is just informative. If you know what they are doing, but not the name of the term, you can still just describe it and get the points. Additionally, make sure that you are familiar with  all the rhetorical devices that are a part of AP Lang!

🎥 Watch: AP Language -   Reading with an Analytical Mind

If it’s not an argumentative essay, what do you put in your thesis?

You state the most important writing choices the author made in order to impact the audience of the work.

Other Tips and Tricks

  • The big idea should show how this prompt applies to today. This will help you write your conclusion. In most language arts classes they teach you to simply restate your points, but not in AP Lang!
  • The first thing you are going to want to do is carefully read through and highlight any strategies you see.
  • Even if TOBI doesn’t work for you, it is a good idea to outline the essay. Even though it takes time, it will end up saving you time in the end because it gives you direction.
  • One of the most useful tools for the introduction is something called Soapstones. In this intro you are introducing the  S peaker,  O ccasion,  A udience,  P urpose,  S ubject,  T one, and  S tyle. (Keep in mind: You do not need to include EVERY ONE). But, most successful essays include a few of them.
  • DON’T SKIM! It will only hurt you in the long run, even if you think it might be saving you time.

how to write an ap lang rhetorical analysis essay

Rhetorical Analysis Example Essay Prompt

The speech below was given at the site of the battle of Gettysburg by president Abraham Lincoln. Lincoln went on to describe his desire to save the union. Read the passage carefully and then in a well-developed essay, analyze the writing choices Lincoln makes to share his message with others. Support your analysis of his rhetoric with specific references from the text.

“Four score and seven years ago our fathers brought forth on this continent a new nation, conceived in Liberty and dedicated to the proposition that all men are created equal. Now we are engaged in a great civil war, testing whether that nation or any nation so conceived and so dedicated, can long endure. We are met on a great battle-field of that war. We have come to dedicate a portion of that field, as a final resting place for those who here gave their lives that that nation might live. It is altogether fitting and proper that we should do this. But, in a larger sense, we can not dedicate—we can not consecrate—we can not hallow—this ground. The brave men, living and dead, who struggled here, have  consecrated it, far above our poor power to add or detract. The world will little note, nor long remember what we say here, but it can never forget what they did here. It is for us the living, rather, to be dedicated here to the unfinished work which they who fought here have thus far so nobly advanced. It is rather for us to be here dedicated to the great task remaining before us—that from these honored dead we take increased devotion to that cause for which they gave the last full measure of devotion—that we here highly resolve that these dead shall not have died in vain—that this nation, under God, shall have a new birth of freedom—and that government of the people, by the people, for the people, shall not perish from the earth.”

Key Terms to Review ( 20 )

© 2024 fiveable inc. all rights reserved., ap® and sat® are trademarks registered by the college board, which is not affiliated with, and does not endorse this website..

Have a language expert improve your writing

Run a free plagiarism check in 10 minutes, generate accurate citations for free.

  • Knowledge Base
  • How to write a rhetorical analysis | Key concepts & examples

How to Write a Rhetorical Analysis | Key Concepts & Examples

Published on August 28, 2020 by Jack Caulfield . Revised on July 23, 2023.

A rhetorical analysis is a type of essay  that looks at a text in terms of rhetoric. This means it is less concerned with what the author is saying than with how they say it: their goals, techniques, and appeals to the audience.

Instantly correct all language mistakes in your text

Upload your document to correct all your mistakes in minutes

upload-your-document-ai-proofreader

Table of contents

Key concepts in rhetoric, analyzing the text, introducing your rhetorical analysis, the body: doing the analysis, concluding a rhetorical analysis, other interesting articles, frequently asked questions about rhetorical analysis.

Rhetoric, the art of effective speaking and writing, is a subject that trains you to look at texts, arguments and speeches in terms of how they are designed to persuade the audience. This section introduces a few of the key concepts of this field.

Appeals: Logos, ethos, pathos

Appeals are how the author convinces their audience. Three central appeals are discussed in rhetoric, established by the philosopher Aristotle and sometimes called the rhetorical triangle: logos, ethos, and pathos.

Logos , or the logical appeal, refers to the use of reasoned argument to persuade. This is the dominant approach in academic writing , where arguments are built up using reasoning and evidence.

Ethos , or the ethical appeal, involves the author presenting themselves as an authority on their subject. For example, someone making a moral argument might highlight their own morally admirable behavior; someone speaking about a technical subject might present themselves as an expert by mentioning their qualifications.

Pathos , or the pathetic appeal, evokes the audience’s emotions. This might involve speaking in a passionate way, employing vivid imagery, or trying to provoke anger, sympathy, or any other emotional response in the audience.

These three appeals are all treated as integral parts of rhetoric, and a given author may combine all three of them to convince their audience.

Text and context

In rhetoric, a text is not necessarily a piece of writing (though it may be this). A text is whatever piece of communication you are analyzing. This could be, for example, a speech, an advertisement, or a satirical image.

In these cases, your analysis would focus on more than just language—you might look at visual or sonic elements of the text too.

The context is everything surrounding the text: Who is the author (or speaker, designer, etc.)? Who is their (intended or actual) audience? When and where was the text produced, and for what purpose?

Looking at the context can help to inform your rhetorical analysis. For example, Martin Luther King, Jr.’s “I Have a Dream” speech has universal power, but the context of the civil rights movement is an important part of understanding why.

Claims, supports, and warrants

A piece of rhetoric is always making some sort of argument, whether it’s a very clearly defined and logical one (e.g. in a philosophy essay) or one that the reader has to infer (e.g. in a satirical article). These arguments are built up with claims, supports, and warrants.

A claim is the fact or idea the author wants to convince the reader of. An argument might center on a single claim, or be built up out of many. Claims are usually explicitly stated, but they may also just be implied in some kinds of text.

The author uses supports to back up each claim they make. These might range from hard evidence to emotional appeals—anything that is used to convince the reader to accept a claim.

The warrant is the logic or assumption that connects a support with a claim. Outside of quite formal argumentation, the warrant is often unstated—the author assumes their audience will understand the connection without it. But that doesn’t mean you can’t still explore the implicit warrant in these cases.

For example, look at the following statement:

We can see a claim and a support here, but the warrant is implicit. Here, the warrant is the assumption that more likeable candidates would have inspired greater turnout. We might be more or less convinced by the argument depending on whether we think this is a fair assumption.

Here's why students love Scribbr's proofreading services

Discover proofreading & editing

Rhetorical analysis isn’t a matter of choosing concepts in advance and applying them to a text. Instead, it starts with looking at the text in detail and asking the appropriate questions about how it works:

  • What is the author’s purpose?
  • Do they focus closely on their key claims, or do they discuss various topics?
  • What tone do they take—angry or sympathetic? Personal or authoritative? Formal or informal?
  • Who seems to be the intended audience? Is this audience likely to be successfully reached and convinced?
  • What kinds of evidence are presented?

By asking these questions, you’ll discover the various rhetorical devices the text uses. Don’t feel that you have to cram in every rhetorical term you know—focus on those that are most important to the text.

The following sections show how to write the different parts of a rhetorical analysis.

Like all essays, a rhetorical analysis begins with an introduction . The introduction tells readers what text you’ll be discussing, provides relevant background information, and presents your thesis statement .

Hover over different parts of the example below to see how an introduction works.

Martin Luther King, Jr.’s “I Have a Dream” speech is widely regarded as one of the most important pieces of oratory in American history. Delivered in 1963 to thousands of civil rights activists outside the Lincoln Memorial in Washington, D.C., the speech has come to symbolize the spirit of the civil rights movement and even to function as a major part of the American national myth. This rhetorical analysis argues that King’s assumption of the prophetic voice, amplified by the historic size of his audience, creates a powerful sense of ethos that has retained its inspirational power over the years.

The body of your rhetorical analysis is where you’ll tackle the text directly. It’s often divided into three paragraphs, although it may be more in a longer essay.

Each paragraph should focus on a different element of the text, and they should all contribute to your overall argument for your thesis statement.

Hover over the example to explore how a typical body paragraph is constructed.

King’s speech is infused with prophetic language throughout. Even before the famous “dream” part of the speech, King’s language consistently strikes a prophetic tone. He refers to the Lincoln Memorial as a “hallowed spot” and speaks of rising “from the dark and desolate valley of segregation” to “make justice a reality for all of God’s children.” The assumption of this prophetic voice constitutes the text’s strongest ethical appeal; after linking himself with political figures like Lincoln and the Founding Fathers, King’s ethos adopts a distinctly religious tone, recalling Biblical prophets and preachers of change from across history. This adds significant force to his words; standing before an audience of hundreds of thousands, he states not just what the future should be, but what it will be: “The whirlwinds of revolt will continue to shake the foundations of our nation until the bright day of justice emerges.” This warning is almost apocalyptic in tone, though it concludes with the positive image of the “bright day of justice.” The power of King’s rhetoric thus stems not only from the pathos of his vision of a brighter future, but from the ethos of the prophetic voice he adopts in expressing this vision.

The conclusion of a rhetorical analysis wraps up the essay by restating the main argument and showing how it has been developed by your analysis. It may also try to link the text, and your analysis of it, with broader concerns.

Explore the example below to get a sense of the conclusion.

It is clear from this analysis that the effectiveness of King’s rhetoric stems less from the pathetic appeal of his utopian “dream” than it does from the ethos he carefully constructs to give force to his statements. By framing contemporary upheavals as part of a prophecy whose fulfillment will result in the better future he imagines, King ensures not only the effectiveness of his words in the moment but their continuing resonance today. Even if we have not yet achieved King’s dream, we cannot deny the role his words played in setting us on the path toward it.

If you want to know more about AI tools , college essays , or fallacies make sure to check out some of our other articles with explanations and examples or go directly to our tools!

  • Ad hominem fallacy
  • Post hoc fallacy
  • Appeal to authority fallacy
  • False cause fallacy
  • Sunk cost fallacy

College essays

  • Choosing Essay Topic
  • Write a College Essay
  • Write a Diversity Essay
  • College Essay Format & Structure
  • Comparing and Contrasting in an Essay

 (AI) Tools

  • Grammar Checker
  • Paraphrasing Tool
  • Text Summarizer
  • AI Detector
  • Plagiarism Checker
  • Citation Generator

The goal of a rhetorical analysis is to explain the effect a piece of writing or oratory has on its audience, how successful it is, and the devices and appeals it uses to achieve its goals.

Unlike a standard argumentative essay , it’s less about taking a position on the arguments presented, and more about exploring how they are constructed.

The term “text” in a rhetorical analysis essay refers to whatever object you’re analyzing. It’s frequently a piece of writing or a speech, but it doesn’t have to be. For example, you could also treat an advertisement or political cartoon as a text.

Logos appeals to the audience’s reason, building up logical arguments . Ethos appeals to the speaker’s status or authority, making the audience more likely to trust them. Pathos appeals to the emotions, trying to make the audience feel angry or sympathetic, for example.

Collectively, these three appeals are sometimes called the rhetorical triangle . They are central to rhetorical analysis , though a piece of rhetoric might not necessarily use all of them.

In rhetorical analysis , a claim is something the author wants the audience to believe. A support is the evidence or appeal they use to convince the reader to believe the claim. A warrant is the (often implicit) assumption that links the support with the claim.

Cite this Scribbr article

If you want to cite this source, you can copy and paste the citation or click the “Cite this Scribbr article” button to automatically add the citation to our free Citation Generator.

Caulfield, J. (2023, July 23). How to Write a Rhetorical Analysis | Key Concepts & Examples. Scribbr. Retrieved September 23, 2024, from https://www.scribbr.com/academic-essay/rhetorical-analysis/

Is this article helpful?

Jack Caulfield

Jack Caulfield

Other students also liked, how to write an argumentative essay | examples & tips, how to write a literary analysis essay | a step-by-step guide, comparing and contrasting in an essay | tips & examples, what is your plagiarism score.

logo-type-white

AP® English Language

3 ap® english language rhetorical essay strategies.

  • The Albert Team
  • Last Updated On: March 1, 2022

3_ap_english_language rhetorical essay strategies

The AP® English Language rhetorical essay can be nightmare inducing for some AP® students, but there is no need for fear. In this exam review we will lay out helpful strategies to get you through the rhetorical essays in no time.

Rhetorical Strategy #1: Dissecting the Prompt

The first rhetorical essay strategy is to dissect the prompt. Understanding what the rhetorical essay wants from you is essential. It is important for you to read the prompt carefully for every essay, but critical reading is even more essential to the rhetorical essay. Your rhetorical prompt that you will be given for the AP® English Language exam will contain two elements. The first element is the concrete task that the prompt is asking of you, which is always to analyze the passage that follows. The second part of the prompt is a more abstract task that is not directly asked for in the prompt, but it is implied. By completely understanding both parts of the prompt, you will be able to give a complete essay that will get you to a higher score.

One example of a prompt from an AP® English Language rhetorical essay is this one from the 2008 exam . The prompt reads:

“In the following passage from The Great Influenza, an account of the 1918 flu epidemic, author John M. Barry writes about scientists and their research. Read the passage carefully. Then, in a well-written essay, analyze how Barry uses rhetorical strategies.”

Here you can see the concrete task that the examiners are asking. They want you to analyze the passage for rhetorical strategies; however, you must figure out what you are analyzing the passage for. That is the more abstract concept that you need to dissect the prompt to find. In the case of Barry’s passage you will need to analyze how he uses rhetorical strategies in order to portray scientific research. We know this, because if you look at the prompt, it specifically states what Barry did in his work, which was to write about science and research. That is your abstract task.

Once you have found your concrete task and your abstract task, a great strategy is to write it down to keep you focused throughout your essay . Using the example above this would look like the following:

Analyze how Barry uses rhetorical strategies in order to portray scientific research.

That sentence is what you must follow when writing your essay, and if you successfully keep to this task, then you will move closer to that high score.

Rhetorical Essay Strategy #2: Stick to the Format

This next rhetorical essay strategy is the key to great organization and structure that will put your test anxiety to bed. There is a simple paragraph structure for the body paragraphs of the AP® English Language rhetorical essay that will allow you to think, write, and score higher, faster. You need to begin each body paragraph with an assertion or claim. That is the point that you are trying to make clear to your audience what you will be proving. A great example of this is from the 2006 AP® English Language rhetorical essay. Below is student 2B’s opening sentence for her first body paragraph.

“The diction of the passage fully relays Hazlitt’s position about money ( student 2B ).”

You can see how the student directly asserts what he or she will be proving in this statement. The next step in constructing your body paragraph is to give one to two pieces of textual evidence. Be sure to state why these quotations relate back to your claim, otherwise they will be deemed irrelevant by the examiners. An example of this is the next sentence in student 2B’s body paragraph about diction. Here, the student brings in elements from the text to support his or her claim about Hazlitt using diction.

“’Rejected’, ‘contempt’, ‘disparaged’, ‘scrutinized’, ‘irksome’, ‘deprived’, ‘assailed’, ‘chagrin’; the endless repetition of such discouragement shows just how emphatically Hazlitt money is requisite for happy life (student 2B).”

The final part of this strategy for conquering the body paragraphs of your rhetorical essays is to end those body paragraphs with a thorough analysis. This is the aspect of the exam where you can put your way of looking at the text into your essay.

An example of this is at the end of student 2B’s body paragraph where he or she states, “The irony of the last sentences is negative, conveying the utter hopelessness of one without money. Though one may have none in life, pitiless men will continue to mock one’s circumstances even after death! (student 2B)”

This analysis of the text adds to the textual examples above and continues to bring in new logic from the student.

When this format of a body paragraph is followed, then it is extremely effective. The essay becomes clear, assertive, and easy to follow for the examiners. Follow this rhetorical essay strategy and you are even closer to getting that 5 on the exam.

Rhetorical Essay Strategy #3: LORA

As you are looking at your AP® English Language rhetorical essay prompt and passage it is important to remember the mnemonic device, LORA. LORA stands for Language, Organization, and Rhetorical Appeals. These elements will help you form your argument.

When you read through your passage you want to think about how the author is utilizing language. Is he or she using figurative language effectively? Is there imagery within the passage? Does the diction of the passage make it more rhetorically persuasive? You should not use all of these, but picking one and analyzing it clearly in one paragraph will keep you focused on how the author uses rhetoric, which is the main task of this essay.

An example of this was in the 2006 AP® English Language rhetorical essay. Student 2A begins his or her first body paragraph with, “One of Hazlitt’s most effective methods of promoting the importance of money is his strong diction (student 2A).” This student begins his or her essay with focusing on diction as how the language is used. He or she then goes on to explain why diction betters Hazlitt’s argument, which is exactly what you must do for your own rhetorical essay.

The organization of the author is the next part of your answer to the prompt. You want to look at how the author organized his or her ideas within the passage to support his or her own argument. By pointing out the organization, or structure, of the work and how it adds to the overall persuasiveness, you will bring two of the three most important elements of rhetoric together in your essay.

After organization you need to look at the rhetoric appeals. You may know them by the names logos, pathos, and ethos. It is suggested that you cover as many of these as possible; however, if time does not permit or if the passage uses one more than the other, then you should focus on one appeal.

One example of using pathos in an essay is from student 2A from the 2006 prompt. “Hazlitt plays on the audience’s heartstrings for more than enough time to convince them of the importance of having money (student 2A).” While it would have been better for the student to directly say that this is pathos, he or she does thoroughly explain the appeal to the passions, or pathos.

Key Takeaways

When taking the AP® English Language rhetoric essay you just need to remember these three rhetorical essay strategies: dissect the prompt, follow the format, and always include LORA. If you can follow them, then you are already on your way to a 5 on the AP® English Language exam .

Let’s put everything into practice. Try this AP® English Language practice question:

Rhetorical Considerations AP® English Language Practice Question

Looking for more AP® English Language practice?

Check out our other articles on AP® English Language .

You can also find thousands of practice questions on Albert.io. Albert.io lets you customize your learning experience to target practice where you need the most help. We’ll give you challenging practice questions to help you achieve mastery of AP® English Language.

Start practicing here .

Are you a teacher or administrator interested in boosting AP® English Language student outcomes?

Learn more about our school licenses here .

Interested in a school license?​

Popular posts.

AP® Physics I score calculator

AP® Score Calculators

Simulate how different MCQ and FRQ scores translate into AP® scores

how to write an ap lang rhetorical analysis essay

AP® Review Guides

The ultimate review guides for AP® subjects to help you plan and structure your prep.

how to write an ap lang rhetorical analysis essay

Core Subject Review Guides

Review the most important topics in Physics and Algebra 1 .

how to write an ap lang rhetorical analysis essay

SAT® Score Calculator

See how scores on each section impacts your overall SAT® score

how to write an ap lang rhetorical analysis essay

ACT® Score Calculator

See how scores on each section impacts your overall ACT® score

how to write an ap lang rhetorical analysis essay

Grammar Review Hub

Comprehensive review of grammar skills

how to write an ap lang rhetorical analysis essay

AP® Posters

Download updated posters summarizing the main topics and structure for each AP® exam.

PrepScholar

Choose Your Test

  • Search Blogs By Category
  • College Admissions
  • AP and IB Exams
  • GPA and Coursework

Expert Guide to the AP Language and Composition Exam

author image

Advanced Placement (AP)

article-71342_640.jpg

With the 2023 AP English Language and Composition exam happening on Tuesday, May 9, it's time to make sure that you're familiar with all aspects of the exam. In this article, I'll give a brief overview of the test, do a deeper dive on each of the sections, discuss how the exam is scored, offer some strategies for studying, and finally wrap up with some essential exam day tips.

Exam Overview

The AP Language and Composition exam tests your rhetorical and composition skills. Essentially, how do authors construct effective arguments in their writing? What tools do they use? How can you use those tools to craft effective writing yourself? That is the essence of rhetorical analysis.

The exam has two parts: the first section is an hour-long, 45 question multiple-choice section. It includes five sets of questions, each based on a passage or passages. In this section, there will be 23-25 rhetorical analysis questions which test your rhetorical skills. There will also be 20-22 writing questions which require you to consider revisions to the texts you're shown.

The second section is free response. It starts with a 15-minute reading period, and then you'll have 120 minutes to write three analytical essays:

  • One essay where you synthesize several provided texts to create an argument
  • One essay where you analyze a nonfiction passage for its rhetorical construction
  • One essay where you create an original argument in response to a prompt.

You will have about 40 minutes to write each essay, but no one will prompt you to move from essay to essay—you can structure the 120 minutes as you wish.

In the next sections I'll go over each section of the exam more closely—first multiple choice, and then free response.

The AP English Language and Composition Multiple-Choice

The multiple-choice section tests you on two main areas. The first is how well you can read and understand nonfiction passages for their use of rhetorical devices and tools. The second is how well you can "think like a writer" and make revisions to texts in composition questions.

You will be presented with five passages, about which you will receive a small amount of orienting information, e.g. "This passage is excerpted from a collection of essays on boating" or "This passage is excerpted from an essay written in 19th-century Haiti." Each passage will be followed by a set of questions.

There are, in general, eight question types you can expect to encounter on the multiple-choice section of the exam. I've taken my examples from the sample questions in the " Course and Exam Description ."

eight-1316133_640.jpg

Magic eight-ball says there are eight types of multiple-choice questions!

Type 1: Reading Comprehension

These questions are focused on verifying that you understood what a certain part of the passage was saying on a concrete, literal level. You can identify these questions from phrases like "according to" "refers," etc. The best way to succeed on these questions is to go back and re-read the part of the passage referred to very carefully.

Comprehension.png

Type 2: Implication

These questions take reading comprehension one step further—they are primarily focused on what the author is implying without directly coming out and saying it. These questions will have a correct answer, though, based on evidence from the passage. Which interpretation offered in the answers does the passage most support? You can identify questions like these from words like "best supported," ‘"implies," "suggests," "inferred," and so on.

implies.png

Type 3: Overall Passage and Author Questions

These questions ask about overall elements of the passage or the author, such as the author's attitude on the issue discussed, the purpose of the passage, the passage's overarching style, the audience for the passage, and so on.

You can identify these questions because they won't refer back to a specific moment in the text. For these questions, you'll need to think of the passage from a "bird's-eye view" and consider what all of the small details together are combining to say.

3overall_passage.png

Type 4: Relationships Between Parts of the Text

Some questions will ask you to describe the relationship between two parts of the text, whether they are paragraphs or specific lines. You can identify these because they will usually explicitly ask about the relationship between two identified parts of the text, although sometimes they will instead ask about a relationship implicitly, by saying something like "compared to the rest of the passage."

4relationship.png

Type 5: Interpretation of Imagery/Figurative Language

These questions will ask you about the deeper meaning or implication of figurative language or imagery that is used in the text. Essentially, why did the author choose to use this simile or this metaphor? What is s/he trying to accomplish?

You can generally identify questions like this because the question will specifically reference a moment of figurative language in the text. However, it might not be immediately apparent that the phrase being referenced is figurative, so you may need to go back and look at it in the passage to be sure of what kind of question you are facing.

5imagery.png

Type 6: Purpose of Part of the Text

Still other questions will ask you to identify what purpose a particular part of the text serves in the author's larger argument. What is the author trying to accomplish with the particular moment in the text identified in the question?

You can identify these questions because they will generally explicitly ask what purpose a certain part of the text serves. You may also see words or phrases like "serves to" or "function."

6purpose_of_part.png

Type 7: Rhetorical Strategy

These questions will ask you to identify a rhetorical strategy used by the author. They will often specifically use the phrase "rhetorical strategy," although sometimes you will be able to identify them instead through the answer choices, which offer different rhetorical strategies as possibilities.

7rhetorical_strategy.png

Type 8: Composition

This is the newest question type, first seen in the 2019/2020 school year. For these questions, the student will need to act as though they are the writer and think through different choices writers need to make when writing or revising text.

These questions can involve changing the order of sentences or paragraphs, adding or omitting information to strengthen an argument or improve clarity, making changes to draw reader attention, and other composition-based choices.

body_composition

Some very important stylish effects going on here.

The AP English Language and Composition Free Response

The free response section has a 15-minute reading period. After that time, you will have 120 minutes to write three essays that address three distinct tasks.

Because the first essay involves reading sources, it is suggested that you use the entire 15-minute reading period to read the sources and plan the first essay. However, you may want to glance at the other questions during the reading period so that ideas can percolate in the back of your mind as you work on the first essay.

Essay One: Synthesis

For this essay, you will be briefly oriented on an issue and then given anywhere from six to seven sources that provide various perspectives and information on the issue. You will then need to write an argumentative essay with support from the documents.

If this sounds a lot like a DBQ , as on the history AP exams, that's because it is! However, this essay is much more argumentative in nature—your goal is to persuade, not merely interpret the documents.

Example (documents not included, see 2022 free response questions ):

body-AP-Literature-synthesis

Essay Two: Rhetorical Analysis

In the second essay, you'll be presented with an excerpt from a nonfiction piece that advances an argument and asked to write an essay analyzing the rhetorical strategies used to construct the passage's argument. You will also be given some orienting information—where the passage was excerpted from, who wrote it, its approximate date, where it was published (if at all), and to whom it was directed.

Example (excerpt not included, see 2022 free response questions ):

body-AP-literature-Question-2

Essay Three: Argument

In the third essay, you will be presented with an issue and asked to write a persuasive essay taking a position on the issue. You will need to support your position with evidence from your "reading, experience, and observations."

body-AP-lit-Question-3

This doesn't look like a very well-constructed argument.

How The AP Language and Composition Exam Is Scored

The multiple-choice section of the exam is worth 45% of your score, and the free-response section is worth the other 55%. So each of the three free-response essays is worth about 18% of your score.

As on other APs, your raw score will be converted to a scaled score of 1-5. This exam has a relatively low 5 rate. Only 10% of test takers received a 5 in 2022 , although 56% of students received a score of 3 or higher.

In terms of how the raw score is obtained, the multiple-choice section is similar to other AP multiple-choice sections: you receive a point for every question you answer correctly, and there is no penalty for guessing.

The grading rubrics for the free-response questions were revamped in 2019. They are scored using analytic rubrics instead of holistic rubrics. For each free-response question, you will be given a score from 0-6. The rubrics assess three major areas:

#1: Thesis (0 to 1 points): Is there a thesis, and does it properly respond to the prompt?

#2: Evidence and Commentary (0 to 4 points): Does the essay include supporting evidence and analysis that is relevant, specific, well organized, and supports the thesis?

#3: Sophistication (0 to 1 points): Is the essay well-crafted and does it show a sufficiently nuanced understanding of the prompt?

Each scoring rubric broadly assesses these three factors. However, each task is also different in nature, so the rubrics do have some differences. I'll go over each rubric—and what it really means—for you here.

Synthesis Essay Rubrics

0 For any of the following:
1

EVIDENCE AND COMMENTARY

0
1 AND
2 AND
3 AND
4 AND

SOPHISTICATION

0
1 Responses that earn this point may demonstrate sophistication of thought and/or a complex understanding of the rhetorical situation by doing any of the following:

dough-196235_640.jpg

Time to synthesize this dough into some cookies.

Rhetorical Analysis Essay Rubrics

0
1 AND
2 AND
3 AND AND
4 AND AND

magnifying-glass-914922_640.png

Examine your texts closely!

Argumentative Essay Rubrics

playing-puppies-790638_640.jpg

The best kind of frenzy is a puppy frenzy!

AP English Language Prep Tips

Unlike its cousin, the AP English Literature and Composition exam, the AP Language and Composition exam (and course) have very little to do with fiction or poetry. So some students used to more traditional English classes may be somewhat at a loss as to what to do to prepare.

Luckily for you, I have a whole slate of preparation tips for you!

Read Nonfiction—In a Smart Way

A major thing you can do to prepare for the AP Lang and Comp exam is to read nonfiction— particularly nonfiction that argues a position , whether explicitly (like an op-ed) or implicitly (like many memoirs and personal essays). Read a variety of non-fiction genres and topics, and pay attention to the following:

  • What is the author's argument?
  • What evidence do they use to support their position?
  • What rhetorical techniques and strategies do they use to build their argument?
  • Are they persuasive? What counterarguments can you identify? Do they address them?

Thinking about these questions with all the reading you do will help you hone your rhetorical analysis skills.

Learn Rhetorical Terms and Strategies

Of course, if you're going to be analyzing the nonfiction works you read for their rhetorical techniques and strategies, you need to know what those are! You should learn a robust stable of rhetorical terms from your teacher, but here's my guide to the most important AP Language and Composition terms .

  • We've compiled a list of 20 rhetorical devices you should know.
  • A heroic individual from Riverside schools in Ohio uploaded this aggressively comprehensive list of rhetorical terms with examples. It's 27 pages long, and you definitely shouldn't expect to know all of these for the exam, but it's a useful resource for learning some new terms.
  • Another great resource for learning about rhetorical analysis and how rhetorical devices are actually used is the YouTube Channel Teach Argument , which has videos rhetorically analyzing everything from Taylor Swift music videos to Super Bowl commercials. It's a fun way to think about rhetorical devices and get familiar with argumentative structures.
  • Finally, a great book—which you might already use in your class—is " They Say, I Say. " This book provides an overview of rhetoric specifically for academic purposes, which will serve you well for AP preparation and beyond.

You also need to practice argumentative and persuasive writing. In particular, you should practice the writing styles that will be tested on the exam: synthesizing your own argument based on multiple outside sources, rhetorically analyzing another piece of writing in-depth, and creating a completely original argument based on your own evidence and experience.

You should be doing lots of writing assignments in your AP class to prepare, but thoughtful, additional writing will help. You don't necessarily need to turn all of the practice writing you do into polished pieces, either—just writing for yourself, while trying to address some of these tasks, will give you a low-pressure way to try out different rhetorical structures and argumentative moves, as well as practicing things like organization and developing your own writing style.

once-upon-a-time-719174_640.jpg

Not the most auspicious start to an argumentative essay.

Practice for the Exam

Finally, you'll need to practice specifically for the exam format. There are sample multiple-choice questions in the " AP Course and Exam Description ," and old free-response questions on the College Board website.

Unfortunately, the College Board hasn't officially released any complete exams from previous years for the AP English Language and Composition exam, but you might be able to find some that teachers have uploaded to school websites and so on by Googling "AP Language complete released exams." I also have a guide to AP Language and Composition practice tests .

Once you're prepped and ready to go, how can you do your best on the test?

Looking for help studying for your AP exam? Our one-on-one online AP tutoring services can help you prepare for your AP exams. Get matched with a top tutor who got a high score on the exam you're studying for!

AP Language and Composition Test Day Tips

Here are four key tips for test-day success.

board-1193334_640.jpg

You are one hundred percent success!

Interact With the Text

When you are reading passages, both on the multiple-choice section and for the first two free-response questions, interact with the text! Mark it up for things that seem important, devices you notice, the author's argument, and anything else that seems important to the rhetorical construction of the text. This will help you engage with the text and make it easier to answer questions or write an essay about the passage.

Think About Every Text's Overarching Purpose and Argument

Similarly, with every passage you read, consider the author's overarching purpose and argument. If you can confidently figure out what the author's primary assertion is, it will be easier to trace how all of the other aspects of the text play into the author's main point.

Plan Your Essays

The single most important thing you can do for yourself on the free-response section of the AP English Language exam is to spend a few minutes planning and outlining your essays before you start to write them.

Unlike on some other exams, where the content is the most important aspect of the essay, on the AP Language Exam, organization, a well-developed argument, and strong evidence are all critical to strong essay scores. An outline will help you with all of these things. You'll be able to make sure each part of your argument is logical, has sufficient evidence, and that your paragraphs are arranged in a way that is clear and flows well.

Anticipate and Address Counterarguments

Another thing you can do to give your free responses an extra boost is to identify counterarguments to your position and address them within your essay. This not only helps shore up your own position, but it's also a fairly sophisticated move in a timed essay that will win you kudos with AP graders.

envelope-392962_640.jpg

Address counterarguments properly or they might get returned to sender!

Key Takeaways

The AP Language and Composition exam tests your rhetorical skills. The exam has two sections.

The first section is an hour-long, 45 question multiple-choice test based on the rhetorical techniques and composition choices.

The second section is a two-hour free-response section (with a 15-minute initial reading period) with three essay questions: one where you must synthesize given sources to make an original argument, one where you must rhetorically analyze a given passage, and one where you must create a wholly original argument about an issue with no outside sources given.

You'll receive one point for every correct answer on the multiple-choice section of the exam, which is worth 45% of your score. The free-response section is worth 55% of your score. For each free-response question, you'll get a score based on a rubric from 0-6. Your total raw score will be converted to a scaled score from 1-5.

Here are some test prep strategies for AP Lang:

#1 : Read nonfiction with an eye for rhetoric #2 : Learn rhetorical strategies and techniques #3 : Practice writing to deploy rhetorical skills #4 : Practice for the exam!

Here are some test-day success tips:

#1 : Interact with each passage you encounter! #2 : Consider every text's overarching purpose and argument. #3 : Keep track of time #4 : Plan your essays #5 : Identify and address counterarguments in your essays.

With all of this knowledge, you're ready to slay the AP English Language and Composition beast!

animal-1299749_640.png

Noble knight, prepare to slay the AP dragon!

What's Next?

Want more AP Lang review? We have a complete collection of released AP Language practice tests , as well as a list of the AP Lang terms you need to know and a guide to the multiple choice section .

Taking the AP Literature exam? Check out our ultimate guide to the AP English Literature test and our list of AP Literature practice tests .

Taking other AP exams? See our Ultimate Guides to AP World History , AP US History , AP Chemistry , AP Biology , AP World History , and AP Human Geography .

Need more AP prep guidance? Check out how to study for AP exams and how to find AP practice tests .

Want to build the best possible college application?   We can help.   PrepScholar Admissions combines world-class admissions counselors with our data-driven, proprietary admissions strategies. We've guided thousands of students to get into their top choice schools, from state colleges to the Ivy League. We know what kinds of students colleges want to admit and are driven to get you admitted to your dream schools. Learn more about PrepScholar Admissions to maximize your chance of getting in:

These recommendations are based solely on our knowledge and experience. If you purchase an item through one of our links, PrepScholar may receive a commission.

Trending Now

How to Get Into Harvard and the Ivy League

How to Get a Perfect 4.0 GPA

How to Write an Amazing College Essay

What Exactly Are Colleges Looking For?

ACT vs. SAT: Which Test Should You Take?

When should you take the SAT or ACT?

Get Your Free

PrepScholar

Find Your Target SAT Score

Free Complete Official SAT Practice Tests

How to Get a Perfect SAT Score, by an Expert Full Scorer

Score 800 on SAT Math

Score 800 on SAT Reading and Writing

How to Improve Your Low SAT Score

Score 600 on SAT Math

Score 600 on SAT Reading and Writing

Find Your Target ACT Score

Complete Official Free ACT Practice Tests

How to Get a Perfect ACT Score, by a 36 Full Scorer

Get a 36 on ACT English

Get a 36 on ACT Math

Get a 36 on ACT Reading

Get a 36 on ACT Science

How to Improve Your Low ACT Score

Get a 24 on ACT English

Get a 24 on ACT Math

Get a 24 on ACT Reading

Get a 24 on ACT Science

Stay Informed

Get the latest articles and test prep tips!

Follow us on Facebook (icon)

Ellen has extensive education mentorship experience and is deeply committed to helping students succeed in all areas of life. She received a BA from Harvard in Folklore and Mythology and is currently pursuing graduate studies at Columbia University.

Ask a Question Below

Have any questions about this article or other topics? Ask below and we'll reply!

Calculate for all schools

Your chance of acceptance, your chancing factors, extracurriculars, how to master the ap english language and composition rhetorical analysis essay.

Hey guys, I'm currently prepping for the AP English Language and Composition exam, and I'm having trouble with the rhetorical analysis essay. What are some tips and resources I should check out to help me ace this part of the exam? Thanks!

Hi there! Rhetorical analysis essays can be challenging, but with focused practice and a solid approach, you can do well on this section. Here are some tips and resources to help you:

1. Understand the rhetorical triangle: When analyzing a text, remember the rhetorical triangle, which consists of the author, audience, and purpose. Familiarize yourself with various rhetorical strategies authors use to engage with their audience and achieve their purpose.

2. Read and annotate: Take time to read and annotate the given passage, underlining or highlighting important rhetorical devices, transitions, and anything else that stands out to you. This will help you form the basis of your analysis.

3. Create an outline: Before writing your essay, jot down the main points you want to cover and organize them in a logical fashion. This will help ensure that your essay is well-structured, flows smoothly, and effectively covers the prompt.

4. Incorporate textual evidence: Always quote or paraphrase specific textual evidence to support your analysis. Don't simply state that rhetorical devices were used; show how they were used by referring to specific instances in the passage.

5. Use precise, concise language: Clearly articulate your analysis in a direct, concise manner. Use specific language and active verbs when analyzing the author's effectiveness.

6. Practice with past prompts: To get better at rhetorical analysis essays, practice by reviewing past AP exam prompts and analyzing various texts. This will help you build up your analytical skillset and familiarize yourself with the types of questions you might encounter.

- College Board's AP English Language and Composition Course and Exam Description (CED): This official College Board document provides a detailed breakdown of the course content, as well as sample exam questions and essays. You can find this on their website.

- CollegeVine: Their blog has multiple articles related to AP English Language and Composition that share tips, strategies, and advice to score well on the exam.

- AP Classroom: If your school has access to the College Board's AP Classroom, there are numerous resources available for practice, including sample essays and prompts.

- Review books: Many reputable companies publish review books specifically tailored to the AP English Language and Composition exam. These books often contain practice questions, summaries of rhetorical strategies, and tips for essay-writing.

About CollegeVine’s Expert FAQ

CollegeVine’s Q&A seeks to offer informed perspectives on commonly asked admissions questions. Every answer is refined and validated by our team of admissions experts to ensure it resonates with trusted knowledge in the field.

IMAGES

  1. ⭐ Ap english language and composition rhetorical analysis essay sample

    how to write an ap lang rhetorical analysis essay

  2. How to Write the AP Lang Rhetorical Essay

    how to write an ap lang rhetorical analysis essay

  3. AP English Language Rhetorical Analysis Essay Outline by Attis Educates

    how to write an ap lang rhetorical analysis essay

  4. AP English Language and Composition Rhetorical Analysis Essay

    how to write an ap lang rhetorical analysis essay

  5. How to Write a Body Paragraph for a Rhetorical Analysis Essay

    how to write an ap lang rhetorical analysis essay

  6. How To Study For Ap Lang Exam

    how to write an ap lang rhetorical analysis essay

VIDEO

  1. Rhetorical Analysis Thesis Tips

  2. I Wrote an Ebook!

  3. AP Lang Rhetorical Analysis Rubric

  4. What is Rhetorical Analysis?

  5. AP Lang Rhetorical Analysis Essay Review and Practice

  6. Rhetorical Analysis Paper Explanation

COMMENTS

  1. How to Write the AP Lang Rhetorical Analysis Essay (With Example)

    The AP Lang Rhetorical Analysis Essay is one of three essays included in the written portion of the AP English Exam. The full AP English Exam is 3 hours and 15 minutes long, with the first 60 minutes dedicated to multiple-choice questions. Once you complete the multiple-choice section, you move on to three equally weighted essays that ask you ...

  2. How to Write the AP Lang Rhetorical Essay

    Tips for Writing the AP Lang Rhetorical Essay. 1. Outline Your Essay Before Writing. One of the most important parts of the AP Lang essays is structuring your essay so that it makes sense to the reader. This is just as important as having good content. For this essay in particular, you'll want to read the passage first and write a brief ...

  3. PDF How to Write a RHETORICAL ANALYSIS ESSAY Step 1: Full Comprehension of

    Step 2: MAD TO WRITE! Follow this process to prepare for any timed rhetorical analysis essay. Some of this is redundant, but this portion has more to do with the actual process of writing an essay, whereas the previous questions are part of simply gaining full comprehension of the text. Main ideas - read to determine what points the speaker makes

  4. Tips on AP Lang rhetorical analysis

    Of course, AP Lang's rhetorical analysis can seem daunting initially, but with the right strategy and plenty of practice, you'll be able to navigate it effectively. ... When you start writing, structure your essay clearly with an intro, body paragraphs, and conclusion. The introduction should provide a brief summary of the text, identify the ...

  5. How to Ace the AP Language Rhetorical Analysis Essay

    In this video, I'll show you how to write the AP English Language rhetorical analysis essay (Q2) step by step using the actual 2017 prompt. Watch me annotate...

  6. AP English Language and Composition: Sample Rhetorical Analysis and

    AP English Language and Composition: Sample Argument Question. The following paragraph is adapted from Mirror for Man, a book written by anthropologist Clyde Kluckhorn in the middle of the twentieth century. Read the passage carefully. Then, write an essay that examines the extent to which the author's characterization of the United States ...

  7. AP Lang Rhetorical Analysis Essay: Tips & Rubric

    The rhetorical analysis essay in the AP Lang exam involves students having to discuss how the authors' contribution to the passage gives a theme or meaning. A synthesis essay involves students creating arguments on the passage or piece of information delivered to them. An argumentative essay requires students to pick a side 'for or against ...

  8. PDF HOW TO WRITE: AP Rhetorical Analysis Paragraphs and Essays

    D:\My Documents\Orlando Teacher docs\AP LANG and COMP\2 Close Reading The Art and Craft of Analysis HOW TO WRITE: AP Rhetorical Analysis Paragraphs and Essays Things you must know in order to accurately analyze a text: 1. SOAPS 2. Rhetorical Strategies a. Appeals (ethos, logos, pathos) b. Style (diction, syntax, details, imagery, tone, etc.) 3.

  9. How to Write a Rhetorical Analysis Essay

    This video explains how to write a rhetorical analysis essay for the AP Lang exam.Ebook Info:Be the first to know when the Coach Hall Writes rhetorical analy... AP English Language and Composition.

  10. Teaching the AP Language and Composition Rhetorical Analysis Essay

    5. Show students how to write a full intro paragraph by typing it out on the spot and thinking aloud. Make students take notes by hand while you are typing. 6. Show students what the body paragraph of a rhetorical analysis essay to look like. I literally type out the paragraph and think aloud while I am writing it.

  11. 2020 AP English Exam: How to Write a Rhetorical Analysis Essay

    In light of The College Board recently announcing that this year's AP English Language and Composition exam will be only one question, a 45-minute rhetorical analysis essay, it would probably be a good idea to freshen up on your essay-writing skills.The benefits of doing this won't stop once high-school ends- knowing how to write rhetorical analysis essays will also be extremely helpful ...

  12. Rhetorical Analysis Essay How-To

    Timing ⏳. You have 40 minutes to complete the rhetorical analysis essay for AP Lang: 12 minutes: Read the text and plan out your essay. (TOBI) 6 minutes: Write your introduction paragraph. 18 minutes: Write 2-3 body paragraphs. 2 minutes: Write a quick conclusion.

  13. 5 Tips to Write a Rhetorical Analysis Essay

    AP Lang Rhetorical Analysis Resources:Check out Mastering Rhetorical Analysis, the new ebook from Coach Hall Writes. This ebook contains all of Coach Hall's ...

  14. AP Lang Rhetorical Analysis Essay

    The rhetorical analysis essay can seem a bit challenging at first, but with practice and a solid strategy, you can ace it! Here are some tips for tackling this essay: Start by reading the text carefully and taking notes on the author's use of rhetorical strategies, such as ethos, pathos, and logos. Jot down specific examples and make note of ...

  15. How to ace rhetorical analysis essays for AP Language?

    Hi there! Rhetorical analysis essays can indeed be challenging, but with the right mindset and a solid strategy, you can excel at them. Here are some tips and guidance to help you improve your essay writing skills: 1. Understand the rhetorical triangle: When analyzing a text, keep the rhetorical triangle in mind (i.e., the relationship between the author, audience, and purpose).

  16. How to Write a Rhetorical Analysis

    A rhetorical analysis is a type of essay that looks at a text in terms of rhetoric. This means it is less concerned with what the author is saying than with how they say it: their goals, techniques, and appeals to the audience. A rhetorical analysis is structured similarly to other essays: an introduction presenting the thesis, a body analyzing ...

  17. How to Write a Rhetorical Analysis Essay

    Check out the UPDATED version here: https://www.youtube.com/watch?v=oU1_rGvAElg&t=109sApologies in advance that this video is SUPER quick. I explain these co...

  18. AP Lang Analysis Essay

    The AP Lang analysis essay, also known as the rhetorical analysis essay, requires you to analyze a text (usually a speech or an article) and explain how the author uses various rhetorical strategies to achieve their purpose. Here are the steps you should follow: 1. Read the prompt: Carefully read the given prompt, which will provide you with ...

  19. 3 AP® English Language Rhetorical Essay Strategies

    Rhetorical Essay Strategy #2: Stick to the Format. This next rhetorical essay strategy is the key to great organization and structure that will put your test anxiety to bed. There is a simple paragraph structure for the body paragraphs of the AP® English Language rhetorical essay that will allow you to think, write, and score higher, faster.

  20. Expert Guide to the AP Language and Composition Exam

    The AP English Language and Composition Multiple-Choice. The multiple-choice section tests you on two main areas. The first is how well you can read and understand nonfiction passages for their use of rhetorical devices and tools. The second is how well you can "think like a writer" and make revisions to texts in composition questions.

  21. PDF HOW TO WRITE: AP Rhetorical Analysis Paragraphs and Essays

    When writing an analysis, it is crucial that you work chronologically through the text. This means that you start at the beginning of the text and work your way through it by discussing what the writer is saying and the effectiveness of the strategies he/she is using at the beginning, middle, and end of the text.

  22. How to master the AP English Language and Composition rhetorical

    Use precise, concise language: Clearly articulate your analysis in a direct, concise manner. Use specific language and active verbs when analyzing the author's effectiveness. 6. Practice with past prompts: To get better at rhetorical analysis essays, practice by reviewing past AP exam prompts and analyzing various texts.

  23. PDF AP English Language and Composition

    The following passage is an excerpt from that speech. Read the passage carefully. Write an essay that analyzes the rhetorical choices Sotomayor makes to convey her message about her identity. In your response you should do the following: • Respond to the prompt with a thesis that analyzes the writer's rhetorical choices.